Απόδειξη Ανισότητας

Συντονιστές: achilleas, emouroukos, silouan

Κω.Κωνσταντινίδης
Δημοσιεύσεις: 42
Εγγραφή: Πέμ Μαρ 22, 2018 5:40 pm

Απόδειξη Ανισότητας

#1

Μη αναγνωσμένη δημοσίευση από Κω.Κωνσταντινίδης » Κυρ Απρ 22, 2018 3:43 pm

Αν οι θετικοί πραγματικοί αριθμοί \chi ,\psi, \zeta
έχουν άθροισμα ίσο με το 1, να δείξετε ότι:

\left ( 1+\frac{1}{\chi } \right )\left ( 1+\frac{1}{\psi } \right )\left ( 1+\frac{1}{\zeta } \right )  \geq 64


Κωνσταντινίδης Κωνσταντίνος

Λέξεις Κλειδιά:
Άβαταρ μέλους
Tolaso J Kos
Δημοσιεύσεις: 5226
Εγγραφή: Κυρ Αύγ 05, 2012 10:09 pm
Τοποθεσία: Λάρισα, Βαρκελώνη
Επικοινωνία:

Re: Απόδειξη Ανισότητας

#2

Μη αναγνωσμένη δημοσίευση από Tolaso J Kos » Κυρ Απρ 22, 2018 5:22 pm

Από την ανισότητα Αρμονικού - Γεωμετρικού Μέσου είναι:
\displaystyle{\left(1+\frac{1}{x}\right)\left(1+\frac{1}{y}\right)\left(1+\frac{1}{z}\right) \geq \left(\frac{3}{\frac{x}{x+1}+\frac{y}{y+1}+\frac{z}{z+1}}\right)^3 } οπότε αρκεί να δειχθεί ότι:

\displaystyle{\frac{x}{x+1}+\frac{y}{y+1}+\frac{z}{z+1} \leq \frac{3}{4}} Όμως:

\displaystyle{\begin{aligned} 
\frac{x}{x+1}+\frac{y}{y+1}+\frac{z}{z+1} &= 1 - \frac{1}{x+1} + 1 - \frac{1}{y+1} + 1 - \frac{1}{z+1} \\  
 &=3 - \left ( \frac{1}{1+x} + \frac{1}{y+1} + \frac{1}{z+1} \right ) 
\end{aligned}}
και άρα από την ανισότητα Αριθμητικού - Αρμονικού Μέσου είναι \displaystyle{\frac{1}{x+1}+\frac{1}{y+1}+\frac{1}{z+1} \geq \frac{3}{\frac{x+1+y+1+z+1}{3}}=\frac{9}{4}} και η ανισότητα έπεται.


Η φαντασία είναι σημαντικότερη από τη γνώση !
\displaystyle{{\color{blue}\mathbf{Life=\int_{birth}^{death}\frac{happiness}{time}\Delta time} }}
Άβαταρ μέλους
george visvikis
Επιμελητής
Δημοσιεύσεις: 13275
Εγγραφή: Παρ Νοέμ 01, 2013 9:35 am

Re: Απόδειξη Ανισότητας

#3

Μη αναγνωσμένη δημοσίευση από george visvikis » Κυρ Απρ 22, 2018 5:25 pm

Είναι σίγουρα στο σωστό φάκελο;


Άβαταρ μέλους
Tolaso J Kos
Δημοσιεύσεις: 5226
Εγγραφή: Κυρ Αύγ 05, 2012 10:09 pm
Τοποθεσία: Λάρισα, Βαρκελώνη
Επικοινωνία:

Re: Απόδειξη Ανισότητας

#4

Μη αναγνωσμένη δημοσίευση από Tolaso J Kos » Κυρ Απρ 22, 2018 5:28 pm

Και μία απόδειξη με την αγαπημένη μου Jensen. Η συνάρτηση f(x) = \ln \left( 1 + \frac{1}{x} \right) είναι κοίλη. Τότε:

\displaystyle{\begin{aligned} 
f(x) + f(y) + f(z) &=\ln \left ( 1 + \frac{1}{x} \right ) + \ln \left ( 1 + \frac{1}{y} \right ) + \ln \left ( 1 + \frac{1}{z} \right ) \\  
 &\geq 3 f \left( \frac{x+y+z}{3} \right ) \\  
 &=3 f \left ( \frac{1}{3} \right ) \\  
 &=3 \ln \left ( 1 + \frac{1}{\frac{1}{3}} \right ) \\ 
 &= 3 \ln 4  
\end{aligned}} Εκθετίζοντας παίρνουμε το ζητούμενο.

Σίγουρα υπάρχουν και άλλες αποδείξεις. Η άσκηση μου είναι γνωστή και οι μόνες αποδείξεις που θυμάμαι αυτή τη στιγμή είναι αυτές οι δύο.


Η φαντασία είναι σημαντικότερη από τη γνώση !
\displaystyle{{\color{blue}\mathbf{Life=\int_{birth}^{death}\frac{happiness}{time}\Delta time} }}
Άβαταρ μέλους
george visvikis
Επιμελητής
Δημοσιεύσεις: 13275
Εγγραφή: Παρ Νοέμ 01, 2013 9:35 am

Re: Απόδειξη Ανισότητας

#5

Μη αναγνωσμένη δημοσίευση από george visvikis » Κυρ Απρ 22, 2018 5:48 pm

Άλλη μία εκτός φακέλου φυσικά.
Από ανισότητα Αριθμητικού-Γεωμετρικού-Αρμονικού μέσου είναι:

\displaystyle \frac{{x + y + z}}{3} \ge \sqrt[3]{{xyz}} \ge \frac{3}{{\frac{1}{x} + \frac{1}{y} + \frac{1}{z}}} \Leftrightarrow \frac{1}{3} \ge \sqrt[3]{{xyz}} \ge \frac{3}{{\frac{1}{x} + \frac{1}{y} + \frac{1}{z}}}, απ' όπου παίρνουμε:

\displaystyle \frac{1}{{xyz}} \ge 27,\frac{1}{x} + \frac{1}{y} + \frac{1}{z} \ge 9 και \displaystyle \frac{1}{{xy}} + \frac{1}{{yz}} + \frac{1}{{xz}} = \frac{{x + y + z}}{{xyz}} = \frac{1}{{xyz}} \ge 27

\displaystyle \left( {1 + \frac{1}{x}} \right)\left( {1 + \frac{1}{y}} \right)\left( {1 + \frac{1}{z}} \right) = 1 + \left( {\frac{1}{x} + \frac{1}{y} + \frac{1}{z}} \right) + \left( {\frac{1}{{xy}} + \frac{1}{{yz}} + \frac{1}{{xz}}} \right) + \frac{1}{{xyz}} \ge 1 + 9 + 27 + 27 \Leftrightarrow

\displaystyle \left( {1 + \frac{1}{x}} \right)\left( {1 + \frac{1}{y}} \right)\left( {1 + \frac{1}{z}} \right) \ge 64. (Η ισότητα ισχύει για \displaystyle x = y = z = \frac{1}{3})
τελευταία επεξεργασία από george visvikis σε Κυρ Απρ 22, 2018 6:04 pm, έχει επεξεργασθεί 1 φορά συνολικά.


Άβαταρ μέλους
Demetres
Γενικός Συντονιστής
Δημοσιεύσεις: 8989
Εγγραφή: Δευ Ιαν 19, 2009 5:16 pm
Τοποθεσία: Λεμεσός/Πύλα
Επικοινωνία:

Re: Απόδειξη Ανισότητας

#6

Μη αναγνωσμένη δημοσίευση από Demetres » Κυρ Απρ 22, 2018 6:04 pm

Μεταφέρθηκε στον σωστό φάκελο.

Δίνω άλλη μία λύση. Επειδή η ισότητα πιάνεται όταν x=y=z=\frac{1}{3} χρησιμοποιώ την ΑΜ-ΓΜ ως εξής:

\displaystyle  1+ \frac{1}{x} = 1 + \frac{1}{3x} + \frac{1}{3x}+ \frac{1}{3x} \geqslant \frac{4}{\sqrt[4]{3^3x^3}}

Άρα

\displaystyle  \left( 1+ \frac{1}{x}  \right)\left( 1+ \frac{1}{y}  \right)\left( 1+ \frac{1}{z}  \right) \geqslant \frac{64}{\sqrt[4]{3^{27} x^3y^3z^3}} \qquad (1)

Όμως από την ΑΜ-ΓΜ έχουμε \displaystyle  \frac{1}{3} = \frac{x+y+z}{3} \geqslant \sqrt[3]{xyz}

Άρα xyz \leqslant \frac{1}{3^3} το οποίο μαζί με την (1) δίνει το ζητούμενο.


Άβαταρ μέλους
silouan
Επιμελητής
Δημοσιεύσεις: 1398
Εγγραφή: Τρί Ιαν 27, 2009 10:52 pm

Re: Απόδειξη Ανισότητας

#7

Μη αναγνωσμένη δημοσίευση από silouan » Κυρ Απρ 22, 2018 9:16 pm

Και άλλη μία προσέγγιση: Από Holder

\displaystyle  \left( 1+ \frac{1}{x}  \right)\left( 1+ \frac{1}{y}  \right)\left( 1+ \frac{1}{z}  \right) \geqslant \left(1+\frac{1}{\sqrt[3]{xyz}}\right)^3

Όμως από την ΑΜ-ΓΜ έχουμε \displaystyle  \frac{1}{3} = \frac{x+y+z}{3} \geqslant \sqrt[3]{xyz}

Άρα xyz \leqslant \frac{1}{3^3} το οποίο δίνει το ζητούμενο.


Σιλουανός Μπραζιτίκος
Mihalis_Lambrou
Επιμελητής
Δημοσιεύσεις: 15762
Εγγραφή: Κυρ Δεκ 21, 2008 2:04 am

Re: Απόδειξη Ανισότητας

#8

Μη αναγνωσμένη δημοσίευση από Mihalis_Lambrou » Δευ Απρ 23, 2018 7:06 am

Κω.Κωνσταντινίδης έγραψε:
Κυρ Απρ 22, 2018 3:43 pm
Αν οι θετικοί πραγματικοί αριθμοί \chi ,\psi, \zeta 
έχουν άθροισμα ίσο με το 1, να δείξετε ότι:

\left ( 1+\frac{1}{\chi } \right )\left ( 1+\frac{1}{\psi } \right )\left ( 1+\frac{1}{\zeta } \right )  \geq 64
Έστω αργά ας δούμε και άλλη μία απόδειξη, την οποία δεν μπορούσα να στείλω χθες γιατί έλειπα όλη μέρα εκτός έδρας.

Έχουμε \frac {1}{3} = \frac {x+y+z}{3} \ge \sqrt [3]{xyz} οπότε το αριστερό μέλος είναι


\displaystayle {\frac {(x+y+z)+ 1 + (xy+yz+zx)+ xy z}{xyz} \ge \frac {(x+y+z)+ 1 }{xyz} + \frac {3\sqrt [3]{x^2y^2z^2} }  {xyz} +\frac {xyz}{xyz}


\displaystayle {= \frac {2}{xyz} + \frac {3 }  {\sqrt [3]{xyz}} +1\ge 2\cdot 27+3\cdot 3+1=64


Math Rider
Δημοσιεύσεις: 137
Εγγραφή: Παρ Απρ 09, 2010 12:40 pm
Τοποθεσία: Πάτρα

Re: Απόδειξη Ανισότητας

#9

Μη αναγνωσμένη δημοσίευση από Math Rider » Δευ Απρ 23, 2018 3:40 pm

Κω.Κωνσταντινίδης έγραψε:
Κυρ Απρ 22, 2018 3:43 pm
Αν οι θετικοί πραγματικοί αριθμοί \chi ,\psi, \zeta 
έχουν άθροισμα ίσο με το 1, να δείξετε ότι:

\left ( 1+\frac{1}{\chi } \right )\left ( 1+\frac{1}{\psi } \right )\left ( 1+\frac{1}{\zeta } \right )  \geq 64
Χρησιμοποιώντας την ανισότητα ΑΜ-ΓΜ έχουμε

\displaystyle{\left ( 1+\frac{1}{x} \right )\left ( 1+\frac{1}{y} \right )\left ( 1+\frac{1}{z} \right )=\left ( 2+\frac{y+z}{x} \right )\left ( 2+\frac{z+x}{y} \right )\left ( 2+\frac{x+y}{z} \right )}\geq

\displaystyle{2\sqrt{\frac{2(y+z)}{x}}\cdot2\sqrt{\frac{2(z+x)}{y}}\cdot2\sqrt{\frac{2(x+y)}{z}}=8\sqrt{8\frac{(x+y)(y+z)(z+x)}{xyz}}\geq 8\sqrt{8\cdot8}=64}.


Νίκος Κ.
Pantelis.N
Δημοσιεύσεις: 26
Εγγραφή: Σάβ Απρ 20, 2019 10:00 pm

Re: Απόδειξη Ανισότητας

#10

Μη αναγνωσμένη δημοσίευση από Pantelis.N » Τετ Μάιος 27, 2020 9:00 pm

Μία γρήγορη με AM-GM:

\prod (1+\frac{1}{x})=\prod \frac{x+1}{x}=\prod \frac{x+x+y+z}{x}\geq \prod \frac{4\sqrt[4]{x^2yz}}{x}=64

Edit: Ευχαριστώ τον Παπαδόπουλο Σταύρο για την επισήμανση του τυπογραφικού λάθους μου.
τελευταία επεξεργασία από Pantelis.N σε Τετ Μάιος 27, 2020 10:38 pm, έχει επεξεργασθεί 1 φορά συνολικά.


ΠΑΠΑΔΟΠΟΥΛΟΣ ΣΤΑΥΡΟΣ
Δημοσιεύσεις: 3600
Εγγραφή: Πέμ Φεβ 27, 2014 9:05 am
Τοποθεσία: ΧΑΛΚΙΔΑ- ΑΘΗΝΑ-ΚΡΗΤΗ

Re: Απόδειξη Ανισότητας

#11

Μη αναγνωσμένη δημοσίευση από ΠΑΠΑΔΟΠΟΥΛΟΣ ΣΤΑΥΡΟΣ » Τετ Μάιος 27, 2020 9:29 pm

Pantelis.N έγραψε:
Τετ Μάιος 27, 2020 9:00 pm
Μία γρήγορη με AM-GM:

\sum (1+\frac{1}{x})=\sum \frac{x+1}{x}=\sum \frac{x+x+y+z}{x}\geq \sum \frac{4\sqrt[4]{x^2yz}}{x}=64
Μάλλον ήθελες να γράψεις

 \prod (1+\frac{1}{x})=\prod \frac{x+1}{x}=\prod \frac{x+x+y+z}{x}\geq \prod \frac{4\sqrt[4]{x^2yz}}{x}=64

το οποίο είναι σωστό.


Απάντηση

Επιστροφή σε “Άλγεβρα - Προχωρημένο Επίπεδο (Juniors)”

Μέλη σε σύνδεση

Μέλη σε αυτήν τη Δ. Συζήτηση: Δεν υπάρχουν εγγεγραμμένα μέλη και 2 επισκέπτες